LSAT and Law School Admissions Forum

Get expert LSAT preparation and law school admissions advice from PowerScore Test Preparation.

User avatar
 Dave Killoran
PowerScore Staff
  • PowerScore Staff
  • Posts: 5852
  • Joined: Mar 25, 2011
|
#88616
Complete Question Explanation
(The complete setup for this game can be found here: lsat/viewtopic.php?f=225&t=35163)

The correct answer choice is (A)

The question stem adds a condition that modifies the original sequence established by the rules. In this instance, the question stem stipulates that V :longline: Z, and thus the original diagram can be re-diagrammed as follows:

G2-Q11-d1.png

Accordingly, we know that M must row at seat 1, and S must row at seat 2. V, Z, and O are in a sequence, but L can float freely among seats 3, 4, 5, and 6.


Answer choice (A): This is the correct answer choice. Answer choice (A) is shown to be true by the discussion of the placement of M and S.

Answer choices (B), (C), (D), and (E): Each of these answer choices could be true, but none of them has to be true and therefore each is incorrect.

Get the most out of your LSAT Prep Plus subscription.

Analyze and track your performance with our Testing and Analytics Package.